LSAT and Law School Admissions Forum

Get expert LSAT preparation and law school admissions advice from PowerScore Test Preparation.

User avatar
 Dave Killoran
PowerScore Staff
  • PowerScore Staff
  • Posts: 5852
  • Joined: Mar 25, 2011
|
#27319
Complete Question Explanation

The correct answer choice is (A)

If F is included in the main-dish recipe, only the 3-4 Dist Possibility #2 shown in setup discussion applies. In that template, G is in the appetizer recipe, and so answer choice (A) is correct.

Alternatively, if F is in the main-dish recipe, then from the second rule N must be included in the appetizer recipe. Since N and G must be in the same recipe per the last rule, G must be included in the appetizer recipe as well.

Get the most out of your LSAT Prep Plus subscription.

Analyze and track your performance with our Testing and Analytics Package.